POTD 2006-04

Table of contents

April 2006

Sunday, April 30

Show off your Calc I skills: 1.

\lim_{n\rightarrow\infty}\;\frac{1}{n^2}\prod_{i=1}^n{\left(n^2+i^2\right)^{\frac{1}{n}}}

2. Does this sum converge:

\sum_{n=1}^{\infty}\, \left| e - \left(1+\frac{1}{n}\right)^{n}\right|

See how you did

Saturday, April 29

from Polytope

You have a perfect 12 hr analog clock. There is an hour hand, a minute hand and a sweep second hand. All the hands have a constant angular velocity. The hands are indistinguishable. Can you tell the time from a strobe photo of the clock?

Friday, April 28

Stolen from Anil's site (http://godel.princeton.edu/wiki/index.php/Main_Page)

Find exact value:

\int_0^1 \frac{\log(1+x)\,dx}{1+x^2}

landen solved this without contour integration but hopes someone can find a good contour. Solution

Bonus Problem, slightly easier and highly related:

Find exact value:

\int_0^{\pi}\log(\sin(x))\,dx

Wednesday, Apr. 26

Evaluate the sum:

\frac{1}{2-\frac{1}{2}}+\frac{1}{2^2-\frac{1}{2^2}}+ \frac{1}{2^4-\frac{1}{2^4}}+\frac{1}{2^8-\frac{1}{2^8}}+\cdots \frac{1}{2^{2^n}-\frac{1}{2^{2^n}}}+\cdots

Solution from landen

Monday, Apr. 24

Find, with proof, all functions f(x) which are defined for real numbers | x | < 1, continuous at x = 0, which satisfy:

f(0)=1,\qquad f(x^2)=\frac{f(x)}{1+x}\quad (|x|<1)

Friday, Apr. 21

from Rustem

Find all strictly increasing, multiplicative functions

f:\mathbb{N}\to\mathbb{R}
(multiplicative here means: f(mn) = f(m) * f(n) when gcd(m,n) = 1)

Bonus Problem

from landen

\mbox{Does this series converge?  }\sum_{n=1}^{\infty}\frac{(2+\sin n)^n}{3^n\,n}

landen plans to attack with continued fractions and irrationality of π. Maybe there is a cute answer. Attack failed. This is a toughie.

Monday, Apr. 17

from ddeerr

Define for positive, real x, not just integers:

x!=\mathrm{factorial}\,(x)=\int_0^\infty u^x\,e^{-u}du
\mbox{Show that }\left(x!\right)^2 > x^x \mbox{ for }x>2\,

Solution

Saturday, Apr. 15

from landen

a_1=\sqrt{2},\ a_2=\sqrt{2}^\sqrt{2},\cdots,\ a_7=\sqrt{2}^{\sqrt{2}^{\sqrt{2}^{\sqrt{2}^{\sqrt{2}^{\sqrt{2}^\sqrt{2}}}}}},\cdots

\mbox{Show that the limit }a_\infty\mbox{ exists and find its value.}

Friday, Apr. 14

from landen

Prove that the series

\frac{1}{1}+\frac{1}{2}-\frac{2}{3}+\frac{1}{4}+\frac{1}{5}-\frac{2}{6}+\frac{1}{7}+\cdots

converges and find its limit. Solution by landen

Wednesday, Apr. 12

from landen

\mbox{A sequence of real numbers }a_0,a_1,a_2\cdots\mbox{ is defined recursively by}
a_0=1,\quad a_{n+1}=\frac{a_n}{1+n\,a_n}
\mbox{Find a general formula for }a_n\,

Monday, Apr. 10

from landen

\mbox{assume:  }\frac{833719}{265381} < \pi < \frac{1146408}{364913}
\mbox{Notice that: }1146408*265381-833719*364913 = 1\,

Use this to show that any better rational approximation to π requires a larger denominator.

Hint: Show that if three positive rational numbers have

\frac{a}{b}<\frac{x}{y}<\frac{c}{d};\mbox{ and }b\,c-a\,d = 1;\mbox{ then: }y>b\mbox{ and }y>d

Saturday, Apr. 8

Rated easy.

Each point in the plane is colored either orange or blue. Prove that one of these colors contains, for each positive value of d, a pair of points at distance d.

Thursday, Apr. 6

landen doesn't know how hard it is.

\mbox{Let }a_1,\,a_2, \cdots\mbox{ be a sequence of positive real numbers,}
\mbox{and let }b_n\mbox{ be the arithmetic mean of } a_1, \,a_2, \cdots, a_n.
\mbox{Prove that if }\sum_{n=1}^\infty\frac{1}{a_n}\mbox{ converges, then so does }\sum_{n=1}^\infty\frac{1}{b_n}.

Wednesday, Apr. 5

landen rated: fairly easy, first idea worked.

\mbox{Let }\theta_n = \arctan(n)\mbox{. Prove that, for } n = 1,2,3,\cdots\,
\theta_{n+1}-\theta_n < \frac{1}{n^2+n}

solution